diff --git a/Analisi matematica 1/Parte teorica/2023-03-23, 24, 28, Proprietà principali della continuità e dei limiti di funzione/main.pdf b/Analisi matematica 1/Parte teorica/2023-03-23, 24, 28, Proprietà principali della continuità e dei limiti di funzione/main.pdf index 3b1cba0..01c8b9b 100644 Binary files a/Analisi matematica 1/Parte teorica/2023-03-23, 24, 28, Proprietà principali della continuità e dei limiti di funzione/main.pdf and b/Analisi matematica 1/Parte teorica/2023-03-23, 24, 28, Proprietà principali della continuità e dei limiti di funzione/main.pdf differ diff --git a/Analisi matematica 1/Parte teorica/2023-03-31, Teoria sulle derivate/main.pdf b/Analisi matematica 1/Parte teorica/2023-03-31, Teoria sulle derivate/main.pdf index f51441f..42d61cb 100644 Binary files a/Analisi matematica 1/Parte teorica/2023-03-31, Teoria sulle derivate/main.pdf and b/Analisi matematica 1/Parte teorica/2023-03-31, Teoria sulle derivate/main.pdf differ diff --git a/Analisi matematica 1/Parte teorica/2023-03-31, Teoria sulle derivate/main.tex b/Analisi matematica 1/Parte teorica/2023-03-31, Teoria sulle derivate/main.tex index d3210d1..ee445cc 100644 --- a/Analisi matematica 1/Parte teorica/2023-03-31, Teoria sulle derivate/main.tex +++ b/Analisi matematica 1/Parte teorica/2023-03-31, Teoria sulle derivate/main.tex @@ -165,4 +165,129 @@ Dimostrare che la derivata sinistra è negativa, e che quella destra è positiva nei casi che hai capito. \end{example} -\end{document} + + \begin{theorem} (di Rolle) + Sia $I = [a, b] \subset \RR$ e sia $f : I \to \RR$ tale che + $f$ sia continua su $I$, che $f(a) = f(b)$ e che $f$ sia derivabile + in $[a, b]$. Allora $\exists \xbar \in (a, b)$ tale che $f'(\xbar) = 0$. + \end{theorem} + + \begin{proof} + Per il teorema di Weierstrass $f$ ammette un punto di massimo $M$ e uno di minimo $m$ in $I$. Se $f(a) = M$ e $f(b) = m$ o viceversa, la + funzione $f$ è costante in $I$, e quindi per ogni punto in $(a, b)$ + la derivata è nulla, dacché $f$ è sempre derivabile. Altrimenti, + sicuramente uno tra il punto di massimo e quello di minimo appartiene + a $(a, b)$. Senza perdita di generalità, si assuma che $\exists x_M \in (a, b)$ tale che $f(x_M) = M$: per + il teorema di Fermat $f'(x_M) = 0$. Analogamente per il caso in cui + $\exists x_m \in (a, b)$ tale che $f(x_m) = m$, da cui la tesi. + \end{proof} + + \begin{theorem} (di Cauchy) + Sia $I = [a, b] \subset \RR$ e siano $f$, $g: I \to \RR$ + continue su $I$ e derivabili in $(a, b)$, con $g'$ non nulla + in $(a, b)$ e $g(a) \neq g(b)$. Allora + $\exists \xbar \in (a, b)$ tale che $\frac{f'(\xbar)}{g'(\xbar)} = \frac{f(b) - f(a)}{g(b)-g(a)}$. + \end{theorem} + + \begin{proof} + Si consideri la funzione $h : I \to \RR$ tale che $h(x) = f(x) - \left(\frac{f(b) - f(a)}{g(b) - g(a)} (g(x) - g(a)) + f(a)\right)$. + Si osserva che $h$, + essendo una somma di funzioni continue su $I$ e derivabili in $(a, b)$, + è anch'essa continua su $I$ e derivabile in $(a, b)$. Inoltre + $h(a) = h(b) = 0$. Quindi, per il teorema di Rolle, $\exists \xbar \in (a, b) \mid h'(\xbar) = 0 \implies \frac{f'(\xbar)}{g'(\xbar)} = \frac{f(b)-f(a)}{g(b)-g(a)}$, + da cui la tesi. + \end{proof} + + \begin{theorem} (di Lagrange) + Sia $I = [a, b] \subset \RR$ e sia $f: I \to \RR$ tale che $f$ + sia continua su $I$ e che $f$ sia derivabile in $(a, b)$. Allora + $\exists \xbar \in (a, b)$ tale che $f'(\xbar) = \frac{f(b) - f(a)}{b-a}$, ossia la cui retta tangente è parallela alla secante + che passa per $(a, f(a))$ e $(b, f(b))$. + \end{theorem} + + \begin{proof} + Si consideri $g(x) = x$, $g$ è continua in $[a, b]$ e derivabile + in $(a, b)$, con derivata sempre non nulla in tale intervallo. + Allora, per il teorema di Cauchy, $\exists \xbar \in (a, b) \mid + f'(\xbar) = \frac{f(b)-f(a)}{b-a}$, da cui la tesi. + \end{proof} + + \begin{proposition} + Sia $I = [a, b] \subset \RR$ e sia $f : I \to \RR$ tale che $f$ + sia continua su $I$ e che $f$ sia derivabile in $(a, b)$, con + derivata non negativa. Allora $f$ è crescente in $[a, b]$. + Analogamente, se la derivata è non positiva, $f$ è decrescente. + \end{proposition} + + \begin{proof} + Senza perdita di generalità si dimostra il caso in cui la derivata + di $f$ in $(a, b)$ è non negativa (altrimenti è sufficiente considerare + $g = -f$). + Si considerino $c < d \in I$. Allora, per il teorema di Lagrange, + $\exists \xbar \in (c, d) \mid f'(c) = \frac{f(d) - f(c)}{d-c} + \implies f(d) - f(c) = \underbrace{f'(c) (d-c)}_{\geq 0} \implies + f(d) \geq f(c)$, ossia che $f$ è crescente in $I$. + \end{proof} + + \begin{remark}\nl + \li L'interpretazione geometrica del teorema di Cauchy, rispetto + a quella di Lagrange, è leggermente più complicata. Si consideri + la curva $\gamma : \RR \to \RR^2$ tale che + $\gamma(t) =(g(t), f(t))$. Si osserva che il coefficiente della + retta tangente in $\xbar$ per $\gamma$ è dato da $\lim_{h \to 0} \frac{f(\xbar + h) - f(\xbar)}{g(\xbar + h) - g(\xbar)}$, che, + sotto le ipotesi del teorema di Cauchy, può essere riscritto + come $\frac{f'(\xbar)}{g'(\xbar)}$. Allora, il teorema di Cauchy + asserisce che esiste un punto della curva $\gamma$ tale per cui + la retta tangente alla curva in quel punto è parallela alla secante + passante per $(g(a), f(a))$ e $(g(b), f(b))$. + \end{remark} + + \begin{exercise} + Dare un esempio di una funzione $f : \RR \to \RR$ crescente e + discontinua $\forall x \in \ZZ$. + \end{exercise} + + \begin{solution} + Si consideri $f(x) = \lfloor x \rfloor$. + \end{solution} + + \begin{exercise} + Si descriva un insieme $X$ tale che i suoi punti di accumulazione + sono $\{\pm 1\}$. + \end{exercise} + + \begin{solution} + Si consideri $X = \{1 + \frac{1}{n}\} \cup \{-1 + \frac{1}{n}\}$. + \end{solution} + + \begin{exercise} + Sia $f : X \to \RRbar$ continua in $\xbar$ e sia $a < f(\xbar)$. + Allora esiste $J$ intorno di $\xbar$ tale che $a < f(x)$ $\forall + x \in J$. + \end{exercise} + + \begin{exercise} + Sia $X \subseteq \RRbar$ e sia $\xbar$ punto di accumulazione di $X$, + $f_1$, $f_2 : X \to \RRbar$. Allora: + + \begin{enumerate}[(i)] + \item Se $f_1 \tendsto{\xbar} +\infty$ e $f_2$ è limitata inferiormente + in un intorno $J$ di $\xbar$, allora $f_1(x) + f_2(x) \tendsto{\xbar} +\infty$. + \item Se $f_1 \tendsto{\xbar} 0$ e $f_2$ è limitata in un intorno + di $\xbar$, allora $f_1 f_2(x) \tendsto{\xbar} 0$. + \item Se $f_1 \tendsto{\xbar} +\infty$ è limitata inferiormente + da una costante positiva $m$ in un intorno $J$ di $\xbar$, allora + $f_1 f_2 \tendsto{\xbar} +\infty$. + \end{enumerate} + \end{exercise} + + \begin{exercise} + Sia $f: \RR \to \RR$ tale che: + + \[ f(x) = \system{x + 2x^2 \sin\left(\frac{1}{x}\right) & \se x \neq 0, \\ 0 & \altrimenti.} \] + + \vskip 0.05in + + Mostrare che $f$ è continua, che $f'(0) = 1$ e che $f'$ non è continua in zero. + \end{exercise} +\end{document} \ No newline at end of file diff --git a/Geometria 1/Teoria spettrale degli endomorfismi/2023-04-03, Es. forma canonica di Jordan reale/main.pdf b/Geometria 1/Teoria spettrale degli endomorfismi/2023-04-03, Es. forma canonica di Jordan reale/main.pdf new file mode 100644 index 0000000..0490bb5 Binary files /dev/null and b/Geometria 1/Teoria spettrale degli endomorfismi/2023-04-03, Es. forma canonica di Jordan reale/main.pdf differ diff --git a/Geometria 1/Teoria spettrale degli endomorfismi/2023-04-03, Es. forma canonica di Jordan reale/main.tex b/Geometria 1/Teoria spettrale degli endomorfismi/2023-04-03, Es. forma canonica di Jordan reale/main.tex new file mode 100644 index 0000000..6ba3f00 --- /dev/null +++ b/Geometria 1/Teoria spettrale degli endomorfismi/2023-04-03, Es. forma canonica di Jordan reale/main.tex @@ -0,0 +1,37 @@ +\documentclass[11pt]{article} +\usepackage{personal_commands} +\usepackage[italian]{babel} + +\title{\textbf{Note del corso di Geometria 1}} +\author{Gabriel Antonio Videtta} +\date{\today} + +\begin{document} + + \maketitle + + \begin{center} + \Large \textbf{Esercitazione: forma canonica di Jordan reale} + \end{center} + + \begin{exercise} + Sia $M \in M(n, \RR)$ tale che $\exists a_1$, ..., $a_k \in \RR$ distinti + tale che: + + \[ (M^2 + a_1^2 I) \cdots (M^2 + a_k^2 I) = 0. \] + + Dimostrare allora che esistono $S$, $A \in M(n, \RR)$ tale che + $M = SA$ con $S$ simmetrica e $A$ antisimmetrica. + \end{exercise} + + \begin{solution} + Per ipotesi, $p(x) = (x^2+a_1^2) \cdots (x^2 + a_k^2) \in \Ker \sigma_M$. + Dal momento che $p(x)$ si scompone in fattori lineari distinti in + $\CC$, $p(x)$ è anche il polinomio minimo di $M$. Si deduce + allora che $M$ è diagonalizzabile, e che i suoi autovalori sono + esattamente $\pm a_1 i$, ..., $\pm a_k i$. Allora la forma + canonica di Jordan reale di $M$ è: + + \[ J = \Matrix{} \] + \end{solution} +\end{document}